7
$\begingroup$

Referring to a question I posted on MS, I post it here, as I didn't get an answer:

By analogy with the Jacobi–Anger expansion, one expects that $e^{iz\cot(x)}$ has a Fourier expansion of the form : $$e^{iz\cot(\theta)}=\sum_{n=-\infty}^{\infty}\Lambda_{n}(z)e^{in\theta}$$ $\Lambda_{n}(z)$ is given by: $$\Lambda_{n}(z)=\frac{1}{\pi}\int_{-\pi/2}^{\pi/2}e^{iz\cot(\theta)-in\theta}d\theta$$ A simple calculation yields: $$\Lambda_{n}(z)=\frac{2}{n\pi}\sin\left(\frac{\pi n}{2} \right )\sum_{m=0}^{\infty}\frac{z^{m}}{m!}\text{F}_{1}\left(-\frac{n}{2};-m,m;1-\frac{n}{2};1,-1 \right )$$

Where $\text{F}_{1}(\alpha;\beta,\beta^{'};\gamma;x,y)$ is the Appell Hypergeometric Function. Now, I have two questions:

1-For a purely imaginary $z$,$\;\;$ $e^{iz\cot(\theta)}$ has essential singularities at $\theta=\pm n\pi$. How is that reflected in the Fourier expansion?

2-Can we express the infinite sum in terms of other special functions?

$\endgroup$

1 Answer 1

1
$\begingroup$

I will try to answer your two questions in reverse order.

$2.$ For real $z$ your Fourier coefficient

$$\Lambda_{n}(z)=\frac{2}{\pi}\int_{0}^{\pi/2}\cos(n\theta-z\cot\theta)\;d\theta$$

can be expressed in terms of elementary functions for even $n$ and in terms of the special functions $I_{m}(z)$ (modified Bessel function of the first kind) and $L_{-m}(z)$ (modified Struve function) for odd $n$. The first few Fourier coefficients are

$$\Lambda_0(z)=e^{-|z|}$$

$$\Lambda_1(z)=\Lambda_{-1}(-z)=z L_{-1}(z)+zL_0(z)-|z|I_0(z)-|z|I_1(z)+2/\pi$$

$$\Lambda_2(z)=\Lambda_{-2}(-z)=e^{-|z|}(z-|z|)$$

For even $n$ the Fourier coefficient $\Lambda_n(z)\equiv 0$ for $nz>0$ [***]

$1.$ The Fourier coefficients, given above along the real $z$-axis, can be continued analytically into the complex-$z$ plane, but not along the imaginary $z$-axis. The branch cut for imaginary $z$ is a direct consequence of the essential singularity you noticed.


[***]

added November 2014 in response to a request for clarification.

since $\Lambda_{n}(z)=\Lambda_{-n}(-z)$, we may take $n>0$. Suppose $n=2m$, $m=1,2,\ldots$ and $z>0$. Then $$ \Lambda_{2m}(z)=\frac{1}{2\pi}{\rm Re}\,\int_{-\pi}^{\pi}e^{im\theta}e^{-iz\cot(\theta/2)}\,d\theta =\frac{1}{2\pi i}{\rm Re}\,\oint_{|\xi|=1^-} \xi^{m-1}e^{-z(1+\xi)/(1-\xi)}\,d\xi=0 $$ For $z<0$ the singularity at $\xi\equiv e^{i\theta}=1$ contributes, so the contour integral does not vanish.

$\endgroup$

Your Answer

By clicking “Post Your Answer”, you agree to our terms of service and acknowledge you have read our privacy policy.

Not the answer you're looking for? Browse other questions tagged or ask your own question.